2000 AMC 10 Problems/Problem 17

Revision as of 08:36, 11 January 2009 by 5849206328x (talk | contribs) (Problem)

Problem

Boris has an incredible coin changing machine. When he puts in a quarter, it returns five nickels; when he puts in a nickel, it returns five pennies; and when he puts in a penny, it returns five quarters. Boris starts with just one penny. Which of the following amounts could Boris have after using the machine repeatedly?

$\mathrm{(A)}$ <dollar/>$3.63$

$\mathrm{(B)}$ <dollar/>$5.13$

$\mathrm{(C)}$ <dollar/>$6.30$

$\mathrm{(D)}$ <dollar/>$7.45$

$\mathrm{(E)}$ <dollar/>$9.07$

Solution

See Also

2000 AMC 10 (ProblemsAnswer KeyResources)
Preceded by
Problem 16
Followed by
Problem 18
1 2 3 4 5 6 7 8 9 10 11 12 13 14 15 16 17 18 19 20 21 22 23 24 25
All AMC 10 Problems and Solutions